Блог пользователя FastestFinger

Автор FastestFinger, 4 года назад, По-английски

1370A - Максимальный НОД

Tutorial
Code

This problem was prepared by the_hyp0cr1t3

1370B - Сжатие массива и НОД

Tutorial
Code

This problem was prepared by Ashishgup and ridbit10

1370C - Игра с числом

Tutorial
Code
Relevant Meme

This problem was prepared by FastestFinger and Ashishgup

1370D - Чет-нечет подпоследовательность

Tutorial
Code

This problem was prepared by Ashishgup

1370E - Сдвиги подпоследовательностей

Tutorial
Code

This problem was prepared by smartnj

1370F2 - Загадана пара вершин (сложная версия)

Tutorial
Code
Relevant Memes

This problem was prepared by FastestFinger and Ashishgup

Meme credits: ridbit10 and the_hyp0cr1t3 and FastestFinger

Разбор задач Codeforces Round 651 (Div. 2)
  • Проголосовать: нравится
  • +587
  • Проголосовать: не нравится

»
4 года назад, # |
  Проголосовать: нравится +25 Проголосовать: не нравится

And the fastest editorial award goes to....

»
4 года назад, # |
  Проголосовать: нравится +15 Проголосовать: не нравится

FastestFinger actually has fastest fingers... Thanks for the fast EDU!

»
4 года назад, # |
Rev. 2   Проголосовать: нравится -16 Проголосовать: не нравится

Wow! Quickest Editorial ever provided. Hats off Ashishgup

  • »
    »
    4 года назад, # ^ |
      Проголосовать: нравится +10 Проголосовать: не нравится

    In case u don't know, once Radewoosh uploaded tutorial before the starting time of the contest ! so technically it's not the fasted tutorial :p

»
4 года назад, # |
Rev. 3   Проголосовать: нравится +74 Проголосовать: не нравится

FastestFinger be like

»
4 года назад, # |
  Проголосовать: нравится -86 Проголосовать: не нравится

CodeForces? More like MathForces

»
4 года назад, # |
  Проголосовать: нравится -10 Проголосовать: не нравится

i got stuck in problem C because i thought it would get tle for checking primality for some numbers as large as 10^9

  • »
    »
    4 года назад, # ^ |
      Проголосовать: нравится 0 Проголосовать: не нравится

    Lol. It would be fine if you just check up to sqrt(n) though.

    • »
      »
      »
      4 года назад, # ^ |
        Проголосовать: нравится 0 Проголосовать: не нравится

      plz post your solution for problem C

      • »
        »
        »
        »
        4 года назад, # ^ |
          Проголосовать: нравится 0 Проголосовать: не нравится

        I think you missed some corner cases. Try cases where n==1, n==2, n==6, and n==18. Nevertheless, here is my submission 84452415 (It's in Java tho)

      • »
        »
        »
        »
        4 года назад, # ^ |
        Rev. 2   Проголосовать: нравится +6 Проголосовать: не нравится

        Just prime factorize the number. If two is not a prime factor which means odd number AG wins. Else if 2 is a pf and its power is 1 then we have to check the powers of other pf's. Let sum of other powers be 'Count'. If its AG turn he will form an odd number with count — 1 powers and divide so that FF get a number 2*(prime number) so only option he has is to divide by the prime number and AG wins. Rest of the cases can be covered in similar way. You can check my submission.

      • »
        »
        »
        »
        4 года назад, # ^ |
          Проголосовать: нравится 0 Проголосовать: не нравится

        try this video solution for clarity: (https://youtu.be/6vVDl0e5jjk)

    • »
      »
      »
      4 года назад, # ^ |
        Проголосовать: нравится 0 Проголосовать: не нравится

      yeah, you are right, i feel so stupid

  • »
    »
    4 года назад, # ^ |
      Проголосовать: нравится 0 Проголосовать: не нравится

    I did that ..but wrong ans

  • »
    »
    4 года назад, # ^ |
      Проголосовать: нравится 0 Проголосовать: не нравится

    Primality checking is O(sqrt(n)), so 10^9 should be fine.

    • »
      »
      »
      4 года назад, # ^ |
      Rev. 3   Проголосовать: нравится 0 Проголосовать: не нравится

      this is my solution 84504888

      • »
        »
        »
        »
        4 года назад, # ^ |
          Проголосовать: нравится 0 Проголосовать: не нравится

        Try out this case: n = 30. Ashishgup should win because if he removes an odd prime, say 3, then FastestFinger has no choice but to remove the other odd prime, 5, leaving Ashishgup with 2 and giving him the win.

        The case where there is more than one odd prime (powers are distinct) and exactly one power of 2, Ashishgup will always win, because he can remove all but one of the odd primes, and Fastestfingers will have no choice but to remove the last odd prime, leaving Ashishgup with 2.

  • »
    »
    4 года назад, # ^ |
      Проголосовать: нравится 0 Проголосовать: не нравится

    I too did got stuck in the same thought, since you can't make a prime table with sieve method and doing brute method will obviously wouldn't work (stupid me :P) so I went back to check if I did some mistake while coming up with a pattern only to waste 20 valuable minutes to understand my foolish mistake in undermining sqrt(n) method of finding prime

  • »
    »
    4 года назад, # ^ |
      Проголосовать: нравится 0 Проголосовать: не нравится

    why to check primality if n is odd then ashish wins because he will divide the number by n

»
4 года назад, # |
Rev. 2   Проголосовать: нравится +5 Проголосовать: не нравится

A Quick editorial with 0 available tutorial at first few minutes. (x

»
4 года назад, # |
  Проголосовать: нравится +1 Проголосовать: не нравится

Can someone help me figure out why 84488594 is failing for problem D? Seems like I have the main idea..

  • »
    »
    4 года назад, # ^ |
      Проголосовать: нравится +3 Проголосовать: не нравится

    Yes please! For the love of god please someone tell me what pretest 10 is :((

  • »
    »
    4 года назад, # ^ |
      Проголосовать: нравится +6 Проголосовать: не нравится

    Try this case: 4 4 1 4 3 2 Your program gives 2 but the correct answer is 3 (I made the same mistake in the contest :P)

    • »
      »
      »
      4 года назад, # ^ |
        Проголосовать: нравится 0 Проголосовать: не нравится

      Thanks. Figured the mistake. Looks like I'm going to be stuck at 1730ish for a while lol.

      • »
        »
        »
        »
        4 года назад, # ^ |
          Проголосовать: нравится +5 Проголосовать: не нравится

        What was your mistake can you tell?

        • »
          »
          »
          »
          »
          4 года назад, # ^ |
            Проголосовать: нравится +1 Проголосовать: не нравится

          Basically, if we only enforce invariant that the smaller subsequence cannot have consecutive elements, you might end up with two consecutive elements inside the bigger subsequence.

          Not sure about you, in the counterexample that is provided my solution picked out 1 2 as the smaller subsequence. But the correct smaller subsequence is 1 3

        • »
          »
          »
          »
          »
          4 года назад, # ^ |
            Проголосовать: нравится +3 Проголосовать: не нравится

          Codeforces Round #651

          Problem A and B

          Problem C

          Problem D

          Problem E check it out guys for complete understanding

        • »
          »
          »
          »
          »
          4 года назад, # ^ |
            Проголосовать: нравится 0 Проголосовать: не нравится

          if k is odd then we can't choose the Nth element to be part of even indices, similarly if k is even we can't choose the Nth element to be part of odd indices

»
4 года назад, # |
  Проголосовать: нравится 0 Проголосовать: не нравится

I almost got Problem C. I thought I followed the right logic. Can someone explain why it doesn't work? (failed on pretest 2)

»
4 года назад, # |
  Проголосовать: нравится 0 Проголосовать: не нравится

I think they create the editorial before creating the problems

»
4 года назад, # |
  Проголосовать: нравится +10 Проголосовать: не нравится

Wowwwww fast editorial! And nice problems!

»
4 года назад, # |
  Проголосовать: нравится -11 Проголосовать: не нравится

Maths only round?

Yes!

»
4 года назад, # |
  Проголосовать: нравится 0 Проголосовать: не нравится

anone can tell how to get the intution behind using binary search in problems like D?

  • »
    »
    4 года назад, # ^ |
      Проголосовать: нравится +22 Проголосовать: не нравится

    Do lots of problems w/ Binary Search tag.

    I've found there are two types of Binary Search Problems: problems you can solve with lower_bound / upper_bound, and problems similar to the one in this contest.

    • »
      »
      »
      4 года назад, # ^ |
        Проголосовать: нравится +9 Проголосовать: не нравится

      exactly .....like suppose you do it with tags mind will look in that direction only .......how to get the idea automatically is what i am asking

      • »
        »
        »
        »
        4 года назад, # ^ |
          Проголосовать: нравится +3 Проголосовать: не нравится

        I find that typically for maximization and minimization problems, I try to see if DP or greedy work. If not, then try binary search and brute force.

        I guess it goes without saying that this requires you to come up with possible solutions and also counterexamples quickly before you start coding.

      • »
        »
        »
        »
        4 года назад, # ^ |
          Проголосовать: нравится +1 Проголосовать: не нравится

        looking at the constraints might also help..since here the constraints are 2*10**5...so we can use O(nlogn)..which usually means we can use any kind of sort in the answer or we can use binary search with O(n) for each iteration of the binary search

    • »
      »
      »
      4 года назад, # ^ |
        Проголосовать: нравится -9 Проголосовать: не нравится

      can you suggest some problems like these ones?

  • »
    »
    4 года назад, # ^ |
    Rev. 2   Проголосовать: нравится +3 Проголосовать: не нравится

    I will share my idea. Whenever you see a monotonic function (step boolean function in problem D is monotonic) you can apply binary search. In fact monotonicity leads to the correctness of binary search.

»
4 года назад, # |
  Проголосовать: нравится 0 Проголосовать: не нравится

@Ashishgup how can you prove that binary search would definitely work in problem D?

  • »
    »
    4 года назад, # ^ |
      Проголосовать: нравится +10 Проголосовать: не нравится

    Define $$$f(x)$$$ as $$$whether$$$ $$$it$$$ $$$is$$$ $$$possible$$$ $$$to$$$ $$$make$$$ $$$answer$$$ $$$<=$$$ $$$x$$$. Clearly, if $$$x$$$ works, then $$$(x + 1)$$$ works as well. You can easily check whether $$$f(x)$$$ is true in $$$O(n)$$$. Hence $$$binary$$$ $$$search$$$ $$$for$$$ $$$answer$$$ works.

»
4 года назад, # |
Rev. 2   Проголосовать: нравится -44 Проголосовать: не нравится

Deleted

»
4 года назад, # |
Rev. 2   Проголосовать: нравится -42 Проголосовать: не нравится

3 reasons why people like Ashishgup should be permanently banned from problemsetting:

1) Round 646

2) Round 648

3) Round 651

  • »
    »
    4 года назад, # ^ |
      Проголосовать: нравится +1 Проголосовать: не нравится

    get a life bro you had only 3 submissions credited to your account , atleast grind more and be in an order to say sth to the problem setters cz right now you are in no position to say so

»
4 года назад, # |
  Проголосовать: нравится +49 Проголосовать: не нравится

Why always asking names as output in C ? instead we can use Yes, No that would be easy and convenient.

»
4 года назад, # |
  Проголосовать: нравится 0 Проголосовать: не нравится

I tried sieve on A.

//Ashishgup gets me again cause upto 10^6..

anyways still better than the horrible 2 no solves streak..

»
4 года назад, # |
  Проголосовать: нравится -33 Проголосовать: не нравится

Today Problem C

»
4 года назад, # |
  Проголосовать: нравится 0 Проголосовать: не нравится

On problem E, does anyone have an intuitive explanation why LCS doesn't work?

I think my submission LCS might recommend that we rotate an odd parity subsequence (which is useless). But my brain is fried so I can't come up with a counterexample.

Failing submission:

84510608

  • »
    »
    4 года назад, # ^ |
      Проголосовать: нравится +7 Проголосовать: не нравится
    10
    1100100110
    0011011001
    

    Answer should be 3 (your program gives 2):

    • Rotate indices $$$0, 3, 4, 5, 8, 9$$$
    • Rotate indices $$$1, 2$$$
    • Rotate indices $$$6, 7$$$

    LCS doesn't work because you have to account for both 101010... subsequences and 010101... subsequences, and LCS causes you to overlap them (indices 4-9 above).

»
4 года назад, # |
  Проголосовать: нравится +3 Проголосовать: не нравится

Can someone explain why we cannot solve C using greedy approach? I tried to maintain 2 different array/sets, and started inserting numbers into them in increasing order, while ensuring that no 2 numbers with consecutive indices land in the same bucket. Final answer should then be min(max(bucket1), max(bucket2))?

  • »
    »
    4 года назад, # ^ |
      Проголосовать: нравится +1 Проголосовать: не нравится

    I think you're referring to D.

    Let's say n=4 k = 4. **Suppose your sorted array (value, index) pairs are as follows : (1, 2) (2, 1) (3, 3) (4, 4)

    Your solution ends up selecting values [1, 4], max=4. We want [2, 3], max=3.

    • »
      »
      »
      4 года назад, # ^ |
      Rev. 2   Проголосовать: нравится +3 Проголосовать: не нравится

      Hey, thanks for your reply. Although I think I might have failed to explain my solution properly. It'll work as follows:

      (1,2), (2,1), (3,3), (4,4)

      Bucket1: empty

      Bucket2: empty

      -> I look at (1,2), insert it in first bucket.

      -> I look at (2,1), try to insert it in first bucket, but it already has an index which is adjacent to index of current element that is being considered, so I insert it in bucket 2.

      -> I look at (3,3), insert it in first bucket

      -> I look at (4,4), try to insert it in first bucket, fail, insert it in second bucket.

      The buckets finally are [1,3] and [2,4], out of which I return 3.

      This is the link to submission in case it helps: 84501650

      • »
        »
        »
        »
        4 года назад, # ^ |
          Проголосовать: нравится 0 Проголосовать: не нравится

        You can try

        6 6
        1 2 1 5 2 1
        

        I think in general, your solution puts the 1 1 1 in one bucket, but the other bucket becomes invalid 2 5 2 has adjacent elements

  • »
    »
    8 месяцев назад, # ^ |
    Rev. 2   Проголосовать: нравится 0 Проголосовать: не нравится
    • Given function is $$$Min(Max(Odd),Max(Even)),$$$ Only one of them can be minimum either max of odd indices or even indices of a subsequence.

    • we can try constructing subsequence, minimizing first parameter of min function which is odd indices of constructed subseuence and ignore even indices.

    • Notice that in this constructed subsequence we might get minimum from even indices and let this value be $$$X$$$,but we ignore bcoz when we do the same construction process for even indices for the value $$$X$$$,we will definetly find this constructed subsequence

    • In short, For a given value $$$X$$$ to occur as minimum, either it should come from odd or even

»
4 года назад, # |
Rev. 4   Проголосовать: нравится +24 Проголосовать: не нравится

Here's my simpler(imo) solution for F: find a node in the path as described in the editorial, note that this also gives you the lenght of the path you have to find. Keep the delimiting nodes of the path u,v at the beginning both equal to the node you found. While dist[u][v] != solution lenght, make a query with all nodes x with min(dist[u][x],dist[v][x])>=(solution lenght-dist[u][v]+1)/2. At least one of these nodes must be in the solution, so with each query you at least halve the remaining lenght of the path. So you will use 1+log2(path lenght)=11 queries at most. I did not get AC, and I think that is only because I did not read correct/incorrect strings, I'm pretty sure this should be AC... Ok, proofed by AC by reading the strings

»
4 года назад, # |
  Проголосовать: нравится 0 Проголосовать: не нравится

How can we prove formally for question D (Odd-Even Subsequence) that the found element while doing binary search always exists in the given array? I tried on some sample tests its working fine, but I am not able to correctly identify why the element does exist in the original array.

  • »
    »
    4 года назад, # ^ |
      Проголосовать: нравится +8 Проголосовать: не нравится

    The boundary where check(x) fails and check(x+1) passes can only exist if x+1 is in the array (otherwise the result of check wouldn't change).

»
4 года назад, # |
Rev. 2   Проголосовать: нравится 0 Проголосовать: не нравится

[deleted]

»
4 года назад, # |
  Проголосовать: нравится +9 Проголосовать: не нравится

What the f---! Binary searching over the answer is the COOLEST idea!! Thanks for the quality questions!

WOW!! I learn so much!

»
4 года назад, # |
  Проголосовать: нравится +14 Проголосовать: не нравится

le editorial usually : come again another day

le editorial with FastestFinger : fast as F boi

»
4 года назад, # |
Rev. 5   Проголосовать: нравится -13 Проголосовать: не нравится

Ignore this comment #Brainfreeze

  • »
    »
    4 года назад, # ^ |
      Проголосовать: нравится +4 Проголосовать: не нравится

    The highest odd factor is 15.

    • »
      »
      »
      4 года назад, # ^ |
        Проголосовать: нравится +5 Проголосовать: не нравится

      My bad! Thank you for the clarification

    • »
      »
      »
      4 года назад, # ^ |
        Проголосовать: нравится 0 Проголосовать: не нравится

      Please help me with the tutorial for D.

      I cannot get what does binary search over the "answer" mean in the key idea of the tutorial?

      I am posting it here since I don't know how to get my doubt to you other than this way.

      • »
        »
        »
        »
        4 года назад, # ^ |
        Rev. 2   Проголосовать: нравится -8 Проголосовать: не нравится

        sort the values in the given array.

        lets index it from [1..n] where A[i] <= A[i+1].

        Let left = 1, right = n.

        Now, take mid = (left+right)/2. and check whether you can get a subsequence with A[mid] as min(max(odd_indices),max(even_indices)), if you can, then your answer lies in [mid,right] else it lies in [1,mid-1].

        EDIT: I am not suggesting to use the sorted for check() function. check() function should be used with original array. I am suggesting Bsearch on sorted array (instead of Bsearching from 1 to 10^9).

        Would recommend you to read about binary search and its applications for more clarity.

        • »
          »
          »
          »
          »
          4 года назад, # ^ |
            Проголосовать: нравится +8 Проголосовать: не нравится

          Sorry, I may not have been able to clearly state my question. I appreciate your reply.

          I know how to implement binary search but was confused with the explanation in the tutorial which asked to binary search the "answer". I assumed the answer meant binary searching the original array and could not understand how is binary search possible for an unsorted array.

          Later, after going through all the comments I found that people were talking about binary searching from 1 to 10e9. After mulling on it for a good amount of time I got that the "x" of the tutorial was indeed to be found using binary search (but not on the array given).

          So, in all, I was able to upsolve the problem

        • »
          »
          »
          »
          »
          4 года назад, # ^ |
            Проголосовать: нравится 0 Проголосовать: не нравится

          If you could kindly say, why are they binary searching on 1 to 10^9 instead of the sorted array? Thanks in advance

          • »
            »
            »
            »
            »
            »
            4 года назад, # ^ |
              Проголосовать: нравится 0 Проголосовать: не нравится

            I don't think there is any specific reason for that. Maybe it is just done to reduce the complexity of explaining the solution.

            • »
              »
              »
              »
              »
              »
              »
              4 года назад, # ^ |
                Проголосовать: нравится 0 Проголосовать: не нравится

              Actually, why is there not a probability for that to give wrong answer?

              • »
                »
                »
                »
                »
                »
                »
                »
                4 года назад, # ^ |
                  Проголосовать: нравится 0 Проголосовать: не нравится

                Why should it give a wrong answer? There isn't probability thing going on here..

      • »
        »
        »
        »
        4 года назад, # ^ |
          Проголосовать: нравится 0 Проголосовать: не нравится

        If we have a $$$monotonic$$$ $$$function$$$ $$$over$$$ $$$a$$$ $$$range$$$ and it is $$$true$$$ $$$for$$$ $$$some$$$ $$$prefix$$$ and $$$false$$$ $$$for$$$ $$$some$$$ $$$suffix$$$ of the range and it is possible to check it's truth value at a point in the range, then you can search for the point where the function's value changes from true to false by $$$binary$$$ $$$search$$$. The idea being that if for some point it is true, then it will definitely be true for values less than that point.

        • »
          »
          »
          »
          »
          4 года назад, # ^ |
            Проголосовать: нравится +8 Проголосовать: не нравится

          Sorry, I may not have been able to clearly state my question and I appreciate your reply.

          I know how to implement binary search but was confused with the explanation in the tutorial which asked to binary search the "answer". I assumed the answer meant binary searching the original array and could not understand how is binary search possible for an unsorted array.

          Later, after going through all the comments I found that people were talking about binary searching from 1 to 10e9. After mulling on it for a good amount of time I got that the "x" of the tutorial was indeed to be found using binary search (but not on the array given).

          So, in all, I was able to upsolve the problem.

          • »
            »
            »
            »
            »
            »
            4 года назад, # ^ |
              Проголосовать: нравится 0 Проголосовать: не нравится

            great! .. actually u could sort the array and do binary search over it as well ..

            • »
              »
              »
              »
              »
              »
              »
              4 года назад, # ^ |
              Rev. 2   Проголосовать: нравится 0 Проголосовать: не нравится

              Yeah, it naturally occured to me after getting the idea of binary search uptil 10e9. In fact, once we sort the array we can fix the index k-1 as the upper bound in our binary search(i.e binary search between 1 to sorted_arr[k-1], where k is the length of the subsequence given as input)

              Just that I was mislead by the tutorial or maybe misread the tutorial.

              Anyways, thanks for your replies.

    • »
      »
      »
      4 года назад, # ^ |
        Проголосовать: нравится 0 Проголосовать: не нравится

      In Editorial of D, while doing binary search at for all <=x elements at odd positions it is understood that all elements at odd position should be <= x but why are we not checking that max element at even places should be greater than x

      ans = min(max(odd pos) ,max(even pos)) = min( x , max(even posotion))

      should't max(even position) be checked to be greated than x

      • »
        »
        »
        »
        4 года назад, # ^ |
          Проголосовать: нравится 0 Проголосовать: не нравится

        If we find an odd sequence {$$$a_1 , a_2 , ..., a_t$$$} and its maximum value is atmost $$$x$$$ , then no matter what the even sequence is we can always say that the $$$answer$$$ $$$<=$$$ $$$x$$$ . Same goes for even sequence

    • »
      »
      »
      4 года назад, # ^ |
      Rev. 2   Проголосовать: нравится 0 Проголосовать: не нравится

      I was solving the interactive problem F1 and got WA on test case 3. When I went through the test case I found that there were more than n-1 pairs of nodes in the test case for all the subtests.

      Please, help me with understanding the tree diagram in such a case since as far as I know : a tree with n nodes has n-1 edges.

      This was my first interactive problem and debugging this would really help me with such problems in future.

      Here, is the link to my submission where you can see the test case 3: Code

      I am posting it here since I don't know how to get my doubt to you( @FastestFinger ) other than this way.

»
4 года назад, # |
  Проголосовать: нравится +11 Проголосовать: не нравится

ashishgup and his team are goats(greatest of all times)

»
4 года назад, # |
Rev. 2   Проголосовать: нравится -14 Проголосовать: не нравится

.

»
4 года назад, # |
  Проголосовать: нравится -25 Проголосовать: не нравится

The following is my recursive DP solution of problem 1370C - Игра с числом

84479320

  • »
    »
    4 года назад, # ^ |
    Rev. 2   Проголосовать: нравится +53 Проголосовать: не нравится

    Fun Fact: naming the array "dp" doesn't make the it a dynamic programming solution

    Fact2: your dp[] value never gets reused.

    • »
      »
      »
      4 года назад, # ^ |
        Проголосовать: нравится 0 Проголосовать: не нравится

      Thanks for the sharing the fun fact. I agree with you than naming an array 'dp' does not make the solution a dynamic programming solution. And I also agree with you that the dp[] value never gets reused for the same test case. However, in the multiple test cases problem, the stored value may be used if the even value $$$n \geq 4$$$ has been solved in previous test case.

      The following is a small test program for this solution which shows that the dp values are reused in the multiple test cases problem.

      Test Program

      • »
        »
        »
        »
        4 года назад, # ^ |
          Проголосовать: нравится +10 Проголосовать: не нравится

        Aaaaand using the array named "dp" over multiple testcases doesn't make it dp either. Just saying.

        • »
          »
          »
          »
          »
          4 года назад, # ^ |
          Rev. 2   Проголосовать: нравится 0 Проголосовать: не нравится

          Note that the array stores the XOR value between (p), the ID of the player to make the move, and the ID of the winner of the game, provided that both players make optimal moves. This solution of a sub-problem when $$$n \geq 4$$$ is an even number can be used by another test case that starts with $$$n$$$ or a larger value. Can you suggest more convenient names to the array and to this approach?

          • »
            »
            »
            »
            »
            »
            4 года назад, # ^ |
              Проголосовать: нравится +1 Проголосовать: не нравится

            Hey man, you can choose any name you like. Even the current name "dp" has nothing wrong in it. You can use the array re-using approach all you want, and yes it will save you at least a little time if you are given such an input that makes you re-use the array. All I'm saying is that it is NOT Dynamic Programming. Re-using a precomputed answer, you can call that memoization at best. You did good, solved the problem, got an AC, congratulations! But for the love of God, don't call it dp.

            • »
              »
              »
              »
              »
              »
              »
              4 года назад, # ^ |
              Rev. 5   Проголосовать: нравится 0 Проголосовать: не нравится

              Thanks for the kind correspondence. I appreciate your concern. I have always thought about dynamic programming to be among the approaches used to solve larger propblems in terms of the stored results of smaller related sub-problems. The following tutorial is very close to this point. DP

              It interesting that removing the unordred map did not change the execution time of the accepted solution. 84540538

»
4 года назад, # |
  Проголосовать: нравится +3 Проголосовать: не нравится

In problem D, I didn't get how binary searching between 1 and 1e9 gives an answer which exists in the array? I understood the logic but isn't there a possibility where for a value x which is doesn't exist in the array satisfies the check condition?

  • »
    »
    4 года назад, # ^ |
      Проголосовать: нравится +1 Проголосовать: не нравится

    There may be cases where some $$$x$$$ satisfies the condition and isn't in the array, but the binary search finds the smallest possible $$$x$$$. It's never optimal to choose an $$$x$$$ not in the array because you can just use the largest value in the subsequence $$$\le x$$$.

  • »
    »
    4 года назад, # ^ |
      Проголосовать: нравится 0 Проголосовать: не нравится

    The loop invariant for the binary search is that check(lo-1, 0) || check(lo-1, 1) is always false, and check(hi, 0) || check(hi, 1) is always true.

    At the end of your binary search, lo == hi. At that point, you know that lo-1 is not a valid x, but lo = hi is valid. So, lo is the smallest possible answer.

»
4 года назад, # |
  Проголосовать: нравится 0 Проголосовать: не нравится

Is there any alternative solution for problem E?

»
4 года назад, # |
Rev. 5   Проголосовать: нравится +4 Проголосовать: не нравится

Explanation for C :

Cases when n is odd or power of 2 is clear , also corner cases n=1 and n=2 can be handled separately.

Now when n is even we can observe that :

let prime factorization of n be 2^a1..3^a2….5^a3….and so on

if a1>1 then

ashish can simple divide n by 3^a2…….5^a3 and so on and give it to fastestfinger. What can fastestfinger do now ? since n is now power of 2. he will have to subtract one and thus ashish will win.

but if a1=1 then

ashish can’t do this since only one two will be left and fastestfinger will just subract 1 and hence win.

So if a1=1 and n has more than one odd divisor ashish will win why ?

for example take n=30 =2*3*5

ashish -> divide n by 3

n=10;

fastestfinger->has only one option that is divide n by 5

n=2;

ashish subtracts 1 and hence wins.

that is ashish will always take (all odd prime factors product except one number (example for n=30 he takes either 3 or 5 and leaves 5 and 3 respectively) so that fastest finger will have to divide by that “except” and then ashish will win by subtracting 1.

else if there is only one odd divisor fastest finger will win .

example n=14.. ashish have to divide by 7 so then n=2 and fastest finger will win.

  • »
    »
    4 года назад, # ^ |
    Rev. 2   Проголосовать: нравится 0 Проголосовать: не нравится

    can you explain n = 36 please

    • »
      »
      »
      4 года назад, # ^ |
        Проголосовать: нравится +1 Проголосовать: не нравится

      For n = 36 : 2,3,4,6,9,12,18 36 = 2*2*3*3 so ashish will div it by 9 Now 36/9 =4 now fastest finger can only sub 1 from it Then it becomes 3 as 3 is odd ashish will win

    • »
      »
      »
      4 года назад, # ^ |
        Проголосовать: нравится +1 Проголосовать: не нравится

      Ashish divides by 9 N=4 fastest finger does -1 no other option Now n becomes 3 Ashish divides by 3 and wins.

    • »
      »
      »
      4 года назад, # ^ |
        Проголосовать: нравится 0 Проголосовать: не нравится

      36. Ashishgup divides by 9 . n = 4 FastestFinger needs to subtract 1 . n = 3 Ashishgup divides by 3 . n = 1 FastestFinger cant do nothing Ashishgup Wins

      Any case where n = (2^k)*(Non-prime-Odd), k!=1 Ashish can divide by that (Non-prime-Odd) Now Fastest is stuck with 2^k, which obviously has no odd multiples, hence forced to subtract Ashish gets 2^k -1 which is always odd and divides it by itself Ashish passes 1 to Fastest and wins.

    • »
      »
      »
      4 года назад, # ^ |
        Проголосовать: нравится +1 Проголосовать: не нравится

      Thank you very much. I understand

»
4 года назад, # |
  Проголосовать: нравится +26 Проголосовать: не нравится

Did anyone else use DSU to solve problem D?

  • »
    »
    4 года назад, # ^ |
      Проголосовать: нравится +4 Проголосовать: не нравится

    Yep, although binary search would probably have been cleaner and faster to implement.

  • »
    »
    4 года назад, # ^ |
      Проголосовать: нравится +1 Проголосовать: не нравится

    Please can you explain general idea for your dsu submission

    • »
      »
      »
      4 года назад, # ^ |
      Rev. 2   Проголосовать: нравится +25 Проголосовать: не нравится

      Yes, I can try.

      Thought Process: We need to fill either the odd or even places with as small elements as possible, to minimise the maximum. Basically, we need k/2 (or k/2+1 for odd positions and odd k) elements which are as small as possible and no two are adjacent.

      To do this, we need to sort the elements while maintaining their previous indices.

      Now, starting from the smallest, we perform the following:

      • Mark its position (original index) as visited
      • If either of the (original) neighbours have been visited, we merge the current position with them using DSU.
      • We calculate the maximum number of elements we are able to take. If it is <k/2 we continue, if it is = k/2 we need to check for few corner cases here whether to continue for one more iteration or to break right now.

      The last visited number is the answer.

      Now, how do we calculate number of elements we are able to take:

      • Maintain a variable to store how many we can take currently.
      • While making a new set, add 1 to it
      • Note that from each set we may take (x+1)/2 elements maximum.
      • While merging two sets, of sizes a and b, subtract their previous contributions from total, and add the contribution of the new merged set.

      There are a few corner cases to handle too here:

      • If k is odd and we can take k/2 elements currently, we must ensure that it contains neither the first, nor the last element.
      • If k is even and we can take k/2 elements currently, we must ensure that it does not contain both the first and the last element.
      • »
        »
        »
        »
        4 года назад, # ^ |
          Проголосовать: нравится +6 Проголосовать: не нравится

        how to make sure u don't take both the first and last element when k is even and what happens when two numbers are equal whose index will u add to dsu

        • »
          »
          »
          »
          »
          4 года назад, # ^ |
          Rev. 2   Проголосовать: нравится +6 Проголосовать: не нравится

          We note that we are forced to take the first (or last) element if and only if:

          1. It has been visited
          2. The size of its set is odd
          • »
            »
            »
            »
            »
            »
            4 года назад, # ^ |
            Rev. 2   Проголосовать: нравится 0 Проголосовать: не нравится

            I didn't thought about that answer is last picked number, and without it I had much more cases to consider, because I was calculating answer. So It was too hard to me to check all cases. And here is my workaround.

            • if subsequence length k is even, I solve twice. 1) for all numbers except last. 2) for all numbers except first. And awaiting size of k/2
            • if subsequence length k is odd, I also solve twice. 1) for all number. awaiting size of (k+1)/2 2) for all numbers except both ends. awaiting size of (k-1)/2.

            For odd size two cases are correspondingly: minimum of maximum is on odd positions, and minimum of maximum is on even positions. Solve is to wait until we can fit required (k/2 or (k+1)/2/...) numbers.

            My code can be easy modified to output subsequence because of this trick. :)

            • »
              »
              »
              »
              »
              »
              »
              4 года назад, # ^ |
                Проголосовать: нравится 0 Проголосовать: не нравится

              I am doing a similar thing by solving for k, and checking if x (lower bound) is coming from odd and even positions.

              But my solution doesn't pass. It would be helpful if you could tell what is wrong, as our approaches are similar (in terms of splitting into odd even). Please have a look at this 84506317

              • »
                »
                »
                »
                »
                »
                »
                »
                4 года назад, # ^ |
                  Проголосовать: нравится 0 Проголосовать: не нравится

                No, your approach is more close to editorial. But with bug. You only checking values that <= x, which is wrong. You need to construct subsequences fairly, honestly, without "number of good elements is enough so it should work". At this moment you may pick first element in both cases.

                • »
                  »
                  »
                  »
                  »
                  »
                  »
                  »
                  »
                  4 года назад, # ^ |
                    Проголосовать: нравится 0 Проголосовать: не нравится

                  (I think my code is not so clear) Let's say I am checking if the values <=x are occupying even positions. Then I require floor(k/2) values such that no two are consecutive (so that there is at least one value for each odd index between them ). Also I make sure that an even index does not consider the first value or the last one (last in case when k is odd). In this way I am not just looking for a good enough frequency, but a set which allows me to select values for the other (odd/even) indices also. This makes me feel that the above is not the bug.

                  If possible provide a test case to tell where something is getting added unnecessarily.

                • »
                  »
                  »
                  »
                  »
                  »
                  »
                  »
                  »
                  4 года назад, # ^ |
                    Проголосовать: нравится +3 Проголосовать: не нравится
                  4 4
                  1 4 4 1
                  
                • »
                  »
                  »
                  »
                  »
                  »
                  »
                  »
                  »
                  4 года назад, # ^ |
                    Проголосовать: нравится 0 Проголосовать: не нравится

                  That makes the bug really clear. Thank you so much. Now it makes perfect sense to construct the subsequence honestly.

        • »
          »
          »
          »
          »
          4 года назад, # ^ |
            Проголосовать: нравится 0 Проголосовать: не нравится

          All the elements (in increasing order of value) only increase the number of numbers we can take. (which is also why the binary search solution works)

          In case of equal numbers, the answer wont change because of the order of taking them because if that number were to be the answer, the last taken occurrence can always lead to the satisfaction of the conditions, even if the occurrences before the last one won't.

  • »
    »
    4 года назад, # ^ |
      Проголосовать: нравится +8 Проголосовать: не нравится

    I used DSU too (and the same approach) but could not find a bug in my code. Ended up thinking DSU approach would never work. Thanks for the proof by AC :P

  • »
    »
    4 года назад, # ^ |
    Rev. 3   Проголосовать: нравится 0 Проголосовать: не нравится

    I too came up with a DSU-based solution to the problem "1370D. Odd-Even Subsequence". If anyone is looking for such implementation kindly have a look.

    84624201

»
4 года назад, # |
  Проголосовать: нравится 0 Проголосовать: не нравится

Please help me out with Problem C[problem:1370C]

https://codeforces.com/contest/1370/submission/84502948

all possible cases which I thought were given right by my code.

  • »
    »
    4 года назад, # ^ |
      Проголосовать: нравится 0 Проголосовать: не нравится

    check for 8 and 18

  • »
    »
    4 года назад, # ^ |
    Rev. 2   Проголосовать: нравится 0 Проголосовать: не нравится

    in case 8 your code print "Ashishgup" , but it should be "FastestFinger"

    explain:

    n = 8 there is no odd divisor for 8 , so Ashishgup will decrease it by one

    n = 7 then FastestFinger will divide it by 7

    n = 1 Ashishgup can't make any move , so FastestFinger win

»
4 года назад, # |
  Проголосовать: нравится +4 Проголосовать: не нравится

Problem C can also be solved by calculating the Grundy number for the given n. It took only 31ms. 84482491

  • »
    »
    4 года назад, # ^ |
      Проголосовать: нравится 0 Проголосовать: не нравится

    Can you explain the intuition behind using grundy number?

    • »
      »
      »
      4 года назад, # ^ |
        Проголосовать: нравится +13 Проголосовать: не нравится

      The Grundy number of a game state is equivalent to the pile size if we think of the state as a single Nim pile. Now, if there is only one Nim pile and it has some stones, first player can just take all the stones. So, in this case if the Grundy number is non-zero, first player wins.

»
4 года назад, # |
  Проголосовать: нравится 0 Проголосовать: не нравится

In problem E, why are we not treating the strings as circular? or is it the case that not treating it circular will not change the answer.?

  • »
    »
    4 года назад, # ^ |
      Проголосовать: нравится +4 Проголосовать: не нравится

    Circular won't change the answer.

  • »
    »
    4 года назад, # ^ |
      Проголосовать: нравится +5 Проголосовать: не нравится

    I don't think it matters if the string is circular or not. My solution was to assign a +1 and -1 to a 01 and 10 respectively and find the maximum and minimum balance(Similar to a parenthesis sequence) and subtract them from each other. A rotation of both strings by an equal amount won't affect the results.

    • »
      »
      »
      4 года назад, # ^ |
        Проголосовать: нравится 0 Проголосовать: не нравится

      Your solution would have very simple implementation. Could you please elaborate on how you noticed the similarity to the parenthesis sequence and how it works? why the answer is the difference of those values?

      • »
        »
        »
        »
        4 года назад, # ^ |
          Проголосовать: нравится 0 Проголосовать: не нравится

        Basically in one move I noticed that you could any neighbouring opposite parenthesis and make them vanish. So you could do this for all neighbouring parenthesis in the same orientation. And I also noticed that you could reduce an uphill sequence in the number of steps equal to the max balance same for the downhill sequence. So basically it narrows down to find max of max balance of all uphill sequences and min of min balance of all downhill sequences.

»
4 года назад, # |
  Проголосовать: нравится +31 Проголосовать: не нравится

»
4 года назад, # |
  Проголосовать: нравится 0 Проголосовать: не нравится

Am I the only one who failed to observe the pattern and brute forced problem C with a recursive solution?

»
4 года назад, # |
  Проголосовать: нравится +3 Проголосовать: не нравится

A pure win/lose recursive solution for C is accepted.

https://codeforces.com/contest/1370/submission/84516783

»
4 года назад, # |
  Проголосовать: нравится 0 Проголосовать: не нравится

How is rating change calculated any ideas?

»
4 года назад, # |
  Проголосовать: нравится +16 Проголосовать: не нравится

Why does $$$binary$$$ $$$search$$$ work in D??

  • »
    »
    4 года назад, # ^ |
      Проголосовать: нравится 0 Проголосовать: не нравится

    Because why it shouldn't ? If we apply binary search on answer , Left = minimum element and Right = maximum element of array. And lets find mid. If we can make a sequence such that at odd indices all the elements are less than mid we can say answer will always be <= mid (Don't care what values are at even position). But there is case possible say k=5 and mid = 5 and the sequence till k-1 that we formed looks like 4 2 5 1 _ and the next elements in original array are greater than 5 (mid). Hence if we're just checking if we can form a sequence where all odd indices are supposed to have elements less than mid, in this case answer doesn't exist but check at even position we have 2, 1 and if we putt any value at last index answer will still be <=mid (in this case 2) . Hence we just have to check if for any mid we can form a sequence where either odd elements are less than mid or even elements are less than mid.

  • »
    »
    4 года назад, # ^ |
    Rev. 3   Проголосовать: нравится +4 Проголосовать: не нравится

    To prove that binary search works, we need to prove that the thing we do binary search over, is monotonic. Sorry if my sentence is incorrect. I'm not native english speaker. For example, if "We do binary search over an array", then the thing is "an array", so we need to prove that the array is monotonic. Proving something is monotonic in straight way is hard. So basic approach is to prove by contradiction. Assume that it is not monotonic then get contradiction.

    What is the thing we do binary search over? (sorry again if it's wrong sentence) We do binary search over "can we make subsequence with cost not exceeding x?" As I said, we will prove it by contradiction. What means that the thing "can we make subsequence with cost not exceeding x?" is monotonic? For each x its value is either "yes" or "no", so monotonic means that it's either: all "no" for each x from zero up to some point then "yes" for each other x, or all "yes" for each x from zero up to some point then "no" up to the end. To prove easily, we pick which one we prove.

    Now proof: Assume that it's non monotonic in the way "no", "yes". Then, to find out how can it be non monotonic, is to take definition of monotonic sequence and make negation. Monotonic means that first all "no", then all "yes". Negation of this statement is that after some x with answer "yes" there is some x' with answer "no". Word "after" means x < x'. But answer "yes" for x means that we can make subsequence with cost not exceeding x, thus it also not exceed x'. Thus answer for x' should be also "yes". Contradiction. Proved.

    Now, to understand it under binary search view, we actually prove that if we pick some x in between and answer is "no", then for all x below it answer is also "no", so we don't need to check them. Contrary, if there would be some "yes" then this yes would be x in assumption, and "no" that we checked by binary search is x'. But we proved that for those x and x' it's impossible. I hope it's all clear now.

»
4 года назад, # |
Rev. 2   Проголосовать: нравится +35 Проголосовать: не нравится

Hello, when I registered for the round my rating was 2102 so I got registered as out of competition but in previous global round I became a candidate master (2088 rating). But cf still shows that I'm out of competition for this round. Can someone tell me if this round will be unrated for me? Thanks in advance!

Upd: Rating has been changed :)

»
4 года назад, # |
  Проголосовать: нравится +14 Проголосовать: не нравится

My solution for F is quite different from the one mentioned in the Editorial. Here it is:

By asking a query over all nodes, find $$$x$$$ and $$$d$$$. Let $$$s = f = x$$$ and $$$target = d$$$. Notice that, $$$target$$$ is the distance between the hidden nodes.

Now, while $$$dist(s, f) < target$$$, let $$$p = \lceil\frac{target-dist(s, f)}{2}\rceil$$$. Let, $$$l_s$$$ be the list of nodes which are at distance $$$p$$$ from $$$s$$$ and they are reachable from $$$s$$$ without using any nodes on the path between current $$$s$$$ and $$$f$$$ (exclusive). Define $$$l_f$$$ in a similar way. Now, we can guarantee that, at least one node $$$x$$$ exists from the union of $$$l_s$$$ and $$$l_f$$$, which is on the path between hidden $$$s$$$ and $$$f$$$. We can find that $$$x$$$ by asking a query over the union of $$$l_s$$$ and $$$l_f$$$. Then, if $$$x$$$ is an element of $$$l_s$$$, replace $$$s$$$ with $$$x$$$. Otherwise ($$$x$$$ is guaranteed to be an element of $$$l_f$$$), replace $$$f$$$ with $$$x$$$.

In each turn of the loop, the distance of $$$s$$$ and $$$f$$$ increases by $$$p$$$. Also, at the end of each loop, current $$$s$$$ and $$$f$$$ lies on the path between hidden $$$s$$$ and $$$f$$$. In the end, $$$s$$$ and $$$f$$$ will achieve the hidden values. Notice that, the distances we find after first query are useless in this solution!

  • »
    »
    4 года назад, # ^ |
    Rev. 4   Проголосовать: нравится 0 Проголосовать: не нравится

    I came up with the same solution (but failed to implement it in the given time, rip, code in case anyone wanted). The nice thing about this solution is that it takes atmost $$$10$$$ queries without having to resort to handling the kinda annoying edge cases which the editorial solution requires.

»
4 года назад, # |
  Проголосовать: нравится 0 Проголосовать: не нравится

Can Anyone please help me understand why my these submissions to B give RTE

84447747

84439096

These are failing on test case : 3 1 3 5 7 9 11

While solution with while loop(84447747) passes on local, but I want to know even in for loop case if second(i<v[0].size()-1) condition fails why code goes inside this loop. Am I missing something basic here.

Would be great if anyone can help, Thanks in Advance!

  • »
    »
    4 года назад, # ^ |
      Проголосовать: нравится +3 Проголосовать: не нравится

    v[0].size() is unsigned. If size is 0 and you subtract 1 from it you get a very large positive number.

»
4 года назад, # |
  Проголосовать: нравится 0 Проголосовать: не нравится

can you do DP for question D? I tried and got nowhere

  • »
    »
    4 года назад, # ^ |
      Проголосовать: нравится +1 Проголосовать: не нравится

    I don't think that DP would help here as any information stored with the state information as the length of some prefix should be sufficient only when there is some information on the length of subsequence actually considered.

    Thus at least 2 dimensions — length of prefix and length of the subsequence are required at least. This would demand a lot of space.

»
4 года назад, # |
  Проголосовать: нравится +1 Проголосовать: не нравится

Can somebody tell me what was your thought process during solving Problem D? How did you get to the solution?

  • »
    »
    4 года назад, # ^ |
      Проголосовать: нравится +1 Проголосовать: не нравится

    In my case , during the whole contest i was thinking about greedy or dp solution. Later i saw in the editorial the word binary search, closed the editorial, thought for a while and got the solution.

    I think remembering to try to use greddy, dp or binary search for minimization/maximisation problems will help.

»
4 года назад, # |
  Проголосовать: нравится +5 Проголосовать: не нравится

FastestFinger In A, why there wasn't this test case 100 1000000 1000000 . . .

My solution would have failed, right?

»
4 года назад, # |
  Проголосовать: нравится 0 Проголосовать: не нравится

Hi everyone, i really need help!!! Can anyone just tell me ,why am i facing RUNTIME ERROR on the 2nd question https://codeforces.com/contest/1370/submission/84517421

  • »
    »
    4 года назад, # ^ |
      Проголосовать: нравится 0 Проголосовать: не нравится

    You are getting an overflow i think for cases where s1.size()==0 or s2.size()==0 so in your for loops s1.size()-1 will actually become 2^63-1 which is quite large and since size of s1 is 0 it would give a runtime error. To view this on line 21 try to print s1.size()-1

    • »
      »
      »
      4 года назад, # ^ |
        Проголосовать: нравится +3 Проголосовать: не нравится

      Thank you very much!!! I got what you are saying and will keep this in mind in future contest.

  • »
    »
    4 года назад, # ^ |
    Rev. 2   Проголосовать: нравится 0 Проголосовать: не нравится

    Hey,

    I also faced same problem turns out s1.size() is unsigned so subtracting 1 gives some big number(2^63-1) so you are entering in for loop.

    here is edited version of your solution which would work soln

»
4 года назад, # |
Rev. 2   Проголосовать: нравится 0 Проголосовать: не нравится

Can somebody please explain how to solve E? Not the key idea, it is very easy, but why finding the maximum sum subarray of that array will work for E, because I cannot understand it from the editorial.

  • »
    »
    4 года назад, # ^ |
      Проголосовать: нравится 0 Проголосовать: не нравится

    You can read Kadane's Algorithm to find the maximum sum subarray.It's a very standard problem. Here, it's the maximum absolute subarray sum so you can first find maximum positive subarray sum, then replace -1s with 1s and vice versa and then again find maximum positive subarray sum and take max of the two.

    • »
      »
      »
      4 года назад, # ^ |
        Проголосовать: нравится 0 Проголосовать: не нравится

      Thanks. I understand how to find the maximum sum subarray, but I don't understand how this relates to this task and why it will work. I'm sorry^ I have written my thoughts in first comment in the not right way, so it must be: "Why find the maximum sum subarray of that array will work for E?".

      • »
        »
        »
        »
        4 года назад, # ^ |
          Проголосовать: нравится +11 Проголосовать: не нравится

        I think the much simpler solution is the one with two sets of posittions. One set contains the 0s in s which are 1s in t, and the other set the 1s of s which are 0s in t.

        With these sets we can more or less simply construct a list of indexes of positions of alternating symbols in s. In each step construct the longest possible list and remove all those elements from the two sets.

        • »
          »
          »
          »
          »
          4 года назад, # ^ |
            Проголосовать: нравится 0 Проголосовать: не нравится

          I tried to do something like this but without sets and it was giving TLE. How I can construct the longest possible list if I have indexes in sets?

          • »
            »
            »
            »
            »
            »
            4 года назад, # ^ |
            Rev. 3   Проголосовать: нравится +6 Проголосовать: не нравится

            Start with the smallest number of both sets, then take the next higher number from the other set, alternating. Since both sets are always of same size this works fairly simple.

            Example 84498652

            This one is more clean 84488036

          • »
            »
            »
            »
            »
            »
            3 года назад, # ^ |
              Проголосовать: нравится 0 Проголосовать: не нравится

            While creating a list of your iterating over all possible value of indexes but you don't have to iterate just do a binary search on the index. Let suppose you choose index ith for s[i]=1 and t[i]=0 then in the set of indexes were s[k]=0 you have to search for just a higher index than ith let say that j. So iterating from ith to jth is not a good idea it will give TLE. Instead you can do a binary search on indexes if you know ith.

            I had also done the same mistake. My Solution for TLE.

            The correct solution of SecondThread who used the same approach with higher(same as lower bound in c++) in treeset in java.

    • »
      »
      »
      4 года назад, # ^ |
        Проголосовать: нравится 0 Проголосовать: не нравится

      Wouldn't the maximum absolute value of the sum of any subarray in a be reduced to maximum length subarray having only 1 or only -1 .

  • »
    »
    4 года назад, # ^ |
      Проголосовать: нравится 0 Проголосовать: не нравится

    I solved E task by simply making brackets sequence from 1 and 0 and finding it's depth.

    • »
      »
      »
      4 года назад, # ^ |
        Проголосовать: нравится 0 Проголосовать: не нравится

      That is clever, because it is super simple code.

      But... how did you come up with the idea?

      I stopped thinking and started implementing when I saw a solution with sets of indexes, and simulating the shift process. Which is obviously more work.

      • »
        »
        »
        »
        4 года назад, # ^ |
        Rev. 2   Проголосовать: нравится +9 Проголосовать: не нравится

        I came up with idea in following way. I did write some bunch of zeros and ones for two strings with equal amount of ones. Then I tried to shift few subsequence, and got idea that picking already matching positions is silly, because we can pick any subsequence. So, then, I threw off my test, and wrote new that doesn't match in each place. I wrote new two strings, and tried to move it as a whole. My hypothesis was that they match. So I tried to beat this hypothesis. Question was to find test where strings doesn't match. It was hard to make a good one, so here what I got:

        10
        1011000110
        0100111001
        

        I still have it because I was testing on it my solution. BTW: always save tests you make, with answers. It will help test a lot. So... When I had this test, I realized, that when I shift it as a whole, some ones are matching and vanish. For a moment I thoughts would it give better result if I shift subsequence? I had feeling that it would not. So I thought about following approach: move all once and remove matching. Repeat until all match. Then, I tried to write simulation using sets, I even wrote most of it, but it had issue. I stucked when I had to find those which will match on next step. And when I was thinking about it, they were matching like a brackets. Also, it's like you bought something and now you can sell it. Which is also like a brackets. So I started to think about brackets. First version that I wrote was calculating place where you can start to make correct brackets sequence, and then summing depths of each closing brackets. When I tested it on samples and on my own test above, result was too high. So I revisit why I am thinking about depth summation, and come up with idea that it's just maximum depth.

        • »
          »
          »
          »
          »
          4 года назад, # ^ |
            Проголосовать: нравится 0 Проголосовать: не нравится

          Thanks!

          • »
            »
            »
            »
            »
            »
            4 года назад, # ^ |
              Проголосовать: нравится 0 Проголосовать: не нравится

            Oh! At first look I thought this is same: https://codeforces.com/blog/entry/79107?#comment-646441

            So I didn't dig into details. But now I see it's much more complicated! Here is my algo in all details.

            My algo in all details
            • »
              »
              »
              »
              »
              »
              »
              4 года назад, # ^ |
                Проголосовать: нравится 0 Проголосовать: не нравится

              I got a visualization like this, consider string of positions in s[] which differ to the ones in t[]: "10111000"

              1: 101    0
              2:    1  0
              3:     10
              

              The first line are the elements we can change with first operation, second with second etc...

              So it is obvious that we need to depth of the bracket sequence. And shifting the sequence is trivial, we just have to consider max(depth)-min(depth).

»
4 года назад, # |
  Проголосовать: нравится -24 Проголосовать: не нравится

C was harder than D. Other than that, nice contest!

»
4 года назад, # |
  Проголосовать: нравится 0 Проголосовать: не нравится

I am unable to find any fault in my code but my code is showing runtime error on test case 2 for problem B. Please help me to get out of this.
Link to my submission is https://codeforces.com/contest/1370/submission/84475986. Thanks in advance for reply :)

  • »
    »
    4 года назад, # ^ |
      Проголосовать: нравится 0 Проголосовать: не нравится

    Did you check the case when the size of your vector is zero?

    • »
      »
      »
      4 года назад, # ^ |
        Проголосовать: нравится 0 Проголосовать: не нравится

      Yes, in that case, program counter does not goes to that loop as i=0 is not less than v.size()-1 as 0>-1

      • »
        »
        »
        »
        4 года назад, # ^ |
        Rev. 3   Проголосовать: нравится +1 Проголосовать: не нравится

        nope v.size() returns unsigned intger , so when the vector is empty it returns 0 and since it is unsinged subrtacting one from it does not make it -1 but makes it 18446744073709551615 . So to avoid this typecast it to signed int first then subtract -1 .

        (int)v.size()-1;
        
»
4 года назад, # |
  Проголосовать: нравится +3 Проголосовать: не нравится

For D, I did the same thing — binary search on x. However, I checked separately for odd and even indices, i.e, the maximum number of valid odd places satisfying <=x condition. Similarly for even indices. The count should be >= ceil(k/2) for odd and >= floor(k/2) for even (when assuming that to provide the answer).

I am unable to understand why my submission: 84506317 fails on test-22. It would be helpful if someone could give a smaller input which fails and/or tell me what mistake I have committed.

»
4 года назад, # |
  Проголосовать: нравится 0 Проголосовать: не нравится

My nlognlogn solution for D passed the pretests but not the system tests. First I sorted all the numbers in an array in ascending order by value. Then I used binary search to get the minimum length prefix of the sorted array which could alone form either the even or the odd indices (when checking whether is it possible I sorted the array in ascending order by index, thus nlognlogn).

»
4 года назад, # |
  Проголосовать: нравится 0 Проголосовать: не нравится

Can anyone explain n = 36 for problem c. How "Ashishgup" wins this game.

  • »
    »
    4 года назад, # ^ |
    Rev. 4   Проголосовать: нравится 0 Проголосовать: не нравится

    36/9 =4 4-1 =3 3/3 =1 So fastest finger has 1. Hence winner is ashish.

  • »
    »
    4 года назад, # ^ |
      Проголосовать: нравится 0 Проголосовать: не нравится

    Ashishgup divides by 9 giving 4 to the other player. Other player can only return 3. Ashishgup can then divide by 3 to give 1 to the other player and win.

  • »
    »
    4 года назад, # ^ |
      Проголосовать: нравится 0 Проголосовать: не нравится

    So 36 = 4 * 9. In his first move Ashishgup divides 36 by 9, so now number is 4. 4 = 2 * 2. Because 4 do not have odd divisors, another player must decrement it, so now number is 3. Than Ashishgup divides 3 by 3 and now number is 1. Now Ashishgup wins because another player cannot do a move.

  • »
    »
    4 года назад, # ^ |
      Проголосовать: нравится 0 Проголосовать: не нравится
    1. Ashishgup divides by 9. N = 4
    2. FastestFinger is forced to subtract 1. N = 3
    3. Ashishgup divides by 3. N = 1
    4. because N = 1, FastestFinger loses -> Ashishgup wins
»
4 года назад, # |
  Проголосовать: нравится 0 Проголосовать: не нравится

https://codeforces.com/contest/1370/submission/84475609 why is my answer wrong on pretest 2??

»
4 года назад, # |
  Проголосовать: нравится 0 Проголосовать: не нравится

Hi, can anyone tell me what is wrong with my approach for E? Instead of finding the min num of 1010s, I tried to find the longest consecutive 1111's that were not alr in correct position. Its failing on pre-test 10 and I'm not too sure why. A counter example would be nice. Thank you in advance! 84521497

»
4 года назад, # |
Rev. 2   Проголосовать: нравится 0 Проголосовать: не нравится

How does number of queries get reduced by 1 after setting lower bound of binary search to ceil(l/2) ??

Consider a tree where there is an edge between ith and (i-1)th node for all i>=2.
And the two hidden nodes are 1 and 2. l in this case l would be 1. The max depth of a node would be 999. So earlier (when lower bound of binary search was 0), we would have searched in [0,999]. Now after changing that we will search in [1,999]. So this would also take 10 queries and hence 12 in total.

What am I missing here??

  • »
    »
    4 года назад, # ^ |
      Проголосовать: нравится +5 Проголосовать: не нравится

    You can also set upper bound to l. In any case I think my solution is more elegant/easier to understand

    • »
      »
      »
      4 года назад, # ^ |
        Проголосовать: нравится 0 Проголосовать: не нравится

      Thanks, figured that out. Just got AC. btw link you mentioned is of editorial. I had a look at your solution though, your code is really elegant. It's niceee.

      • »
        »
        »
        »
        4 года назад, # ^ |
          Проголосовать: нравится 0 Проголосовать: не нравится

        It is a link to another comment of mine in this page, which explains my solution, for some reason today I decided to use rust, since it won't be rated for me anyway, so code ended up being somewhat long...

»
4 года назад, # |
  Проголосовать: нравится 0 Проголосовать: не нравится

I have a question about C — Number Game: What if n is 18, shouldn't FastestFinger win? Here is my reasoning:

Ashishgup — 18 / 9 = 2 FastestFinger — 2 — 1 = 1 Ashishgup — 1

Since Ashishgup has 1, FastFinger should win. Is there something I am doing wrong since my answer WA on test case 2 when n = 18

  • »
    »
    4 года назад, # ^ |
      Проголосовать: нравится 0 Проголосовать: не нравится

    Ashishgup will play optimally so he will divide by 3 and not 9

  • »
    »
    4 года назад, # ^ |
      Проголосовать: нравится 0 Проголосовать: не нравится

    Ashish — 18/3 = 6
    Fastest — 6/3 = 2
    Ashish — 2-1 = 1
    Fastest — ....

    Ashish wins

  • »
    »
    4 года назад, # ^ |
      Проголосовать: нравится 0 Проголосовать: не нравится

    AshishGup being one of the smartest(True Story) will divide by 3 in the first move making n=6 now if FastestFinger makes it 5 Ashish divides by 5 and wins otherwise is FastestFinger makes it 3 Ashish divides by 3 and wins.

»
4 года назад, # |
Rev. 2   Проголосовать: нравится 0 Проголосовать: не нравится

Someone please help me with the tutorial for D.

I cannot get what does binary search over the "answer" mean in the key idea of the tutorial?

»
4 года назад, # |
  Проголосовать: нравится 0 Проголосовать: не нравится

Hey guys, I need help on problem D. The approach I found seems correct, but it fails on TC 10.

Here it goes: Let's say the optimal answer is x. Obviously, x should be equal to an element in our array. Otherwise, assuming we found an optimal answer, we could decrease it until it reaches $$$min(max(s_1,s_3,s_5,…),max(s_2,s_4,s_6,…))$$$ (our answer). Let's now say that $$$x = a_l$$$.

We can try to binary search $$$l$$$ in the following way: Say in our current step we are at $$$mid$$$. We sort the array in ascending order and we mark the first $$$mid$$$ elements (using true/false values). Then, we apply this marking to the initial arrangement. Now, we will try to create a desirable sequence and try to fit as many marked elements as possible at either odd or even indices (I will call them from now on subsequences). Say we can squeeze them in the even indices. Then, we get $$$min(max(s_1,s_3,s_5,…),max(s_2,s_4,s_6,…)) = max(s2,s4,s6,...) <= a_{mid}$$$, as $$$a_{mid}$$$ is marked as well. Hence, we get a valid sequence (and we can decrease $$$mid$$$ in our binary search).

One way to see if we have an answer is to get the maximum marked elements we can get in odd or even indices. If we have two adjacent marked elements, we can only use one of them in our subsequence, because if both of them are included in the even one, then we can't have any element in the odd index between them). So, we can use dynamic programming in the following way:

$$$DP_i$$$ denotes the max number of marked elements that there exist in even or odd indices in the first $$$i$$$ elements of our sequence. Here is our recurrence relationship: $$$DP_{i + 1} = max(DP_i, DP_{i - 2} + 1, if i is marked)$$$. That means, we can either ignore our current element and get the answer from the previous one or (if it's marked) we can include it, but we can't use the previous element, hence the $$$DP_{i - 2}$$$ term.

The maximum subsequence we can form will be equal to $$$DP_n$$$. If it's bigger than the desired length of the subsequence (more about it in a moment), then we can return true on the binary search predicate. But there is one more technicality we need to resolve. What should we do with $$$k$$$? Let's have a look.

*If $$$k$$$ is even, then it doesn't matter whether we are trying to fill an even or an odd subsequence, as they should have the same size. Therefore, we can try and find the min $$$l$$$ such as we squeeze exactly $$$k / 2$$$ elements in an either odd or even subsequence.

*However, if $$$k$$$ is odd, then the even subsequence will be one less. So, we have 2 cases: *if the first element in the array is not marked, then we include it in our odd subsequence and solve the problem on the interval $$$(2...n)$$$ with $$$(k - 1) / 2$$$. *if on the other hand it's marked, then it's best to include it on the odd subsequence. Then, we see that our problem is reduced to the above one (ie. interval $$$(2...n)$$$ with $$$(k - 1) / 2$$$).

Here is my submission: https://codeforces.com/contest/1370/submission/84494240

»
4 года назад, # |
Rev. 2   Проголосовать: нравится +3 Проголосовать: не нравится

However, we can solve C with DP.

Suppose that dp[n] = 1, is First player wins if we start with n, 2 if second wins.

Then dp[1] = 2, dp[2] = 1. For all odd N, dp[N] = 1. If n mod 2 = 0 and n > 2, we cant subtract 1 from n(because n becomes odd and second player wins).

So we can only divide N. lets find all odd divisors of N greater then 1.

Suppose we find divisor f which is odd. When if dp[n / f] = 2, it means that dp[n] = 1.

Dont forget to save dp states in recursion! Code: https://codeforces.com/contest/1370/submission/84458094.

»
4 года назад, # |
  Проголосовать: нравится +20 Проголосовать: не нравится

In problem E , can someone prove that why is it never optimal to include any indices $$$i$$$ such that $$$s_i = t_i $$$ ? I couldn't prove this by myself.

  • »
    »
    4 года назад, # ^ |
    Rev. 2   Проголосовать: нравится 0 Проголосовать: не нравится

    I don't know how to prove it. But there always exist a way to get minimum number of operations by ignoring the place $$$s_i=t_i$$$.(The way is mention in tutorial).So we can ignore the $$$s_i=t_i$$$ place.

  • »
    »
    4 года назад, # ^ |
      Проголосовать: нравится 0 Проголосовать: не нравится

    Taking those indices would always require atleast 2 cyclic shifts in some cases where not taking them does in lesser steps. Also, we don't even need more than one cyclic shift in any optimal answer for each subsequence as we could split string s into many subsequences. This proves that taking indices of equal parity will only increase the number of steps.

  • »
    »
    4 года назад, # ^ |
      Проголосовать: нравится 0 Проголосовать: не нравится

    In editorial there is no proof. Ashishgup It's bad habit I think to not include proof but place some substitution instead. It it relies on very non-intuitive thing that $$$s_i = t_i$$$ can be ignored. I don't know how to prove it. Ashishgup if you have original proof though, I would like to see it, because here is what I got, and it's complicated enough. I added some redundant explanations just to make it rigorous. Short version could be made, but it is proof not substitution.

    There are some moments in editorial that I would call mistakes. First, is that subsequences must be of the form 0101. And part with "assume $$$c \geq 0$$$ (otherwise we can interchange 1 and -1)".

    Instead of proving that $$$s_i = t_i$$$ could be ignored, I'll prove that algorithm from editorial is correct, and it indeed find optimal solution. I'll use some facts and observation from editorial, but add more details, more facts and fix things I dislike. And fact that you can omit matching positions will be as a corollary of the proof.

    Alright.

    horrible wall of text
»
4 года назад, # |
  Проголосовать: нравится 0 Проголосовать: не нравится

»
4 года назад, # |
  Проголосовать: нравится +53 Проголосовать: не нравится

CaseForces :)

»
4 года назад, # |
  Проголосовать: нравится 0 Проголосовать: не нравится

Hi, please tell me why I am getting runtime error in it. https://codeforces.com/contest/1370/submission/84441745

»
4 года назад, # |
  Проголосовать: нравится 0 Проголосовать: не нравится

can someone plz explain editorial solution of E

»
4 года назад, # |
  Проголосовать: нравится +13 Проголосовать: не нравится

This was kind of Mathsforces.. But liked the simplicity of problem statements and toughness of implementing them..

»
4 года назад, # |
Rev. 2   Проголосовать: нравится 0 Проголосовать: не нравится

Why I am getting RTE on this 84528459 submission on problem E? Upd: I get it

»
4 года назад, # |
  Проголосовать: нравится 0 Проголосовать: не нравится

can anyone pls explain what does cur ^= 1; do???(code of problem D)

  • »
    »
    4 года назад, # ^ |
    Rev. 2   Проголосовать: нравится 0 Проголосовать: не нравится

    It's cur = cur xor 1

    They;re using cur as a boolean representing whether they can take the next number or not, as they cannot take consecutive numbers from the array in for the odd/even sub sequences. In this context, cur ^= 1 is just negating the current value of it.

»
4 года назад, # |
  Проголосовать: нравится +11 Проголосовать: не нравится

I have a non-binary search solution for D that instead uses Union-Find

Code: 84528808

Key idea: Iterate through the numbers smallest to largest, and soon as you can create a valid sequence for either the odd or even indices with the numbers so far, you have found your answer

Explanation:

First, sort a, but keep each number connected to it's original index Then go from smallest to largest on the number. For each number, get it's index in the original array. Check whether the numbers above/below it have been reached yet, and if they have, merge then current number into those groups. We can increase the length of a sequence possible if it does not merge with a group of odd size. (Each group is a sequence of sequential indices that have all already been considered)

Once we have a valid sequence that is long enough, our answer is the biggest number in a that we have examined.

Care needs to be taken with numbers near the beginning/end of the original array since the first number cannot be part of an even indexed sequence, and the last number cannot be part of one of the sequences depending on whether k is even or odd.

»
4 года назад, # |
Rev. 2   Проголосовать: нравится 0 Проголосовать: не нравится

Here's an alternative $$$n\log(n)$$$ solution for D I found during the contest, but I'm stuck at a step, can anyone help?

The task can be reduced (using odd/even casework) to finding the subsequence $$$s$$$ of length $$$m$$$ of array $$$b$$$ which does not contain two consecutive elements from $$$b$$$, and has smallest maximum value among all such subsequences. This can be achieved by sorting the elements in $$$b$$$ in ascending order. Then, we iteratively add each element to a ordered list of "intervals". For example, if $$$b = [b_1, b_2, b_3, b_4, b_5]$$$, and the ascending order is $$$b_1, b_5, b_2, b_4, b_3$$$, then the list looks like:

$$$[(1, 1)]$$$
$$$[(1, 1), (5, 5)]$$$
$$$[(1, 2), (5, 5)]$$$
$$$[(1, 2), (4, 5)]$$$
$$$[(1, 5)]$$$

You can do these calculations by using a version of binary search on this list to find the right position. Then by tracking how the intervals change it is possible to determine what is the maximum length of a non-consecutive subsequence using only the numbers so far, and we stop when this is $\geq m$.

The problem is, it takes linear time to perform insertion/deletion operations on this list if implemented as a vector, which makes the overall runtime quadratic in $$$n$$$. Does anyone know of a data structure which can avoid this problem?

Edit: Found the comment above mine which uses Union-Find :)

»
4 года назад, # |
  Проголосовать: нравится -13 Проголосовать: не нравится

Why there are N pairs of inputs in problem F1 and F2?

»
4 года назад, # |
  Проголосовать: нравится +19 Проголосовать: не нравится

I have an alternative solution to E:

Lets define a new string a as follows: we delete all indexes such that si = ti, and in the remaining ones we append si at the end of a.

Then a looks like 001110.

We need to take subsequences like 01010101 or 101010, and delete them.

We can see that it can looks like a bracket sequence, but like 01 and 10 can't be deleted in the same operation, we will change each 01 by '(' ')' and each 10 by '[' ']' greedily.

So, now 0110 look like [](), 110010 like (())() and 00111100 like (())[[]].

We can delete any balaced subsequence of depth 1 which contains only () or [].

Then the optimal solution is just the sum of the depth of the strings formed by all '(' and ')', plus the depth of the string formed by '[' and ']', we can compute this easily with the classical algorithm used to check if a bracket sequence is balanced.

We can't do it in less operations, because in each operation we can reduce the depth of "()", or "[]" by at most 1.

  • »
    »
    4 года назад, # ^ |
      Проголосовать: нравится +9 Проголосовать: не нравится

    I was thinking about the same approach in contest, but I could not figure out how to convert the given string into a valid bracket sequence? For example, the conversions which you mentioned "now 0110 look like [](), 110010 like (())() and 00111100 like (())[[]]." As you can see, sometimes you are putting ( and another times you are putting [. How do you make that decision?

    • »
      »
      »
      4 года назад, # ^ |
      Rev. 2   Проголосовать: нравится +21 Проголосовать: не нравится

      You can do it greedily. Basically, you can maintain a counter, and add 1 whenever you encounter a 0, add 1 to the counter, and whenever you encounter a 0, do counter--. This means that whenever the counter is positive, we will have the [] type of brackets (it is not optimal to have both at the same time). Whenever it is negative we will have the () type. So, the answer would be max(counter) — min(counter), because we want the maximum depth of [] and also the max depth of () (which is negative so we have a — sign).

      I solved it with this idea during the contest here.

      It turns out this is exactly the same as taking the absolute maximum subarray sum, as the editorial.

    • »
      »
      »
      4 года назад, # ^ |
        Проголосовать: нравится 0 Проголосовать: не нравится

      I just do the following, maintain two counters, one for 01 and one for 10, when i found a 1, check if i had a 0 non-matched previously, and put a ')', else put a '[', similarly when i found a 0, check if there is a 1 non-matched previously, put a ']', else put a '('. The prove of this greedy is non trivial, but it can be reduced to subarray of maximal absolute sum like Charis02 says above.

»
4 года назад, # |
  Проголосовать: нравится 0 Проголосовать: не нравится

Can anyone explain problem c for value 54? How it will be "Ashishgup"?

  • »
    »
    4 года назад, # ^ |
      Проголосовать: нравится 0 Проголосовать: не нравится

    54, 6, 2, 1. 54 = 2 * 27. So keep only 1 odd number (3) in 27. FF will have no choice other than taking that. Then take a 1.

»
4 года назад, # |
  Проголосовать: нравится 0 Проголосовать: не нравится

Can anyone explain why should we use binary search in problem D! I have read the tutorial again and again and still not been able to understand why should binary search works fine here ? How can we say that the problem is monotonic (the necessary condition for using binary search)?

  • »
    »
    4 года назад, # ^ |
    Rev. 2   Проголосовать: нравится 0 Проголосовать: не нравится

    Let $$$p$$$ be our answer. Now, let $$$can(x)$$$ be a function that will give us whether our answer can be $$$\leq{x}$$$.

    Now notice that for values $$$\geq{p}$$$ our function will give us true value, for values $$$<{p}$$$ it will give us false. That's why it is monotonic. I hope you understand.

»
4 года назад, # |
  Проголосовать: нравится 0 Проголосовать: не нравится

Whenever there is "Determine the winner of the game if both of them play optimally." for me it sucks.

»
4 года назад, # |
  Проголосовать: нравится 0 Проголосовать: не нравится

Can anyone prove that We can ignore all the indices where si=ti as it is never optimal to include those indices in a rotation. in E.

»
4 года назад, # |
  Проголосовать: нравится 0 Проголосовать: не нравится

Ashishgup, in problem E, i have tried the approach of first finding longest occurring of 0101 or 1010 pattern in mismatched string. But I am getting wrong ans in pretest 10. Can anyone help me figure out what am i doing wrong?

Here is my submission for problem E. 84505487

»
4 года назад, # |
Rev. 2   Проголосовать: нравится +4 Проголосовать: не нравится
»
4 года назад, # |
Rev. 2   Проголосовать: нравится 0 Проголосовать: не нравится

Deleted

»
4 года назад, # |
  Проголосовать: нравится 0 Проголосовать: не нравится

Can someone explain how can we solve D using dp strategy?

»
4 года назад, # |
  Проголосовать: нравится +15 Проголосовать: не нравится

Binary search: ruining careers since 1962.

»
4 года назад, # |
  Проголосовать: нравится 0 Проголосовать: не нравится

Nice problem D!

I did not know that binary search could be utilized like this, was really informative. Thanks for the problem!

»
4 года назад, # |
  Проголосовать: нравится 0 Проголосовать: не нравится

Can anyone explain me in problem D solution why we are taking odd and even indices of original array while we have to take indices of new subsequence of length k.

»
4 года назад, # |
Rev. 2   Проголосовать: нравится 0 Проголосовать: не нравится

In problem D,can't we apply Binary Search on the range of min to max elements of the array rather than 1 to 1e9?Will it give correct or wrong answer.Can anyone please clear this up for me?

  • »
    »
    4 года назад, # ^ |
      Проголосовать: нравится 0 Проголосовать: не нравится

    You can use that too, since all possible answers are actually the elements of the array. But why bother. But it can be done.

»
4 года назад, # |
  Проголосовать: нравится +1 Проголосовать: не нравится

Can anyone help me in F2 why it is giving WA https://codeforces.com/contest/1370/submission/84557344

»
4 года назад, # |
Rev. 2   Проголосовать: нравится 0 Проголосовать: не нравится
#include<bits/stdc++.h>
using namespace std;
string game(int n,int c)
{
    if(n==1&&c==1)
    return "FastestFinger";
    if(n==1&&c==2)
    return "Ashishgup";
    if(n%2!=0&&c==1)
    return game(n/n,2);
    if(n%2!=0&&c==2)
    return game(n/n,1);
    if(n%2==0&&c==1)
    return game(n-1,2);
    if(n%2==0&&c==2)
    return game(n-1,1);
}
int main()
{
    int T,n,c;
    cin>>T;
    while(T--)
    {
        cin>>n;
        c=1;
        cout<<game(n,1)<<"\n";
    }
}

can pls someone tell me whats wrong with my code?
  • »
    »
    4 года назад, # ^ |
      Проголосовать: нравится +1 Проголосовать: не нравится

    When n is even you are letting the other player win by making n odd. Consider the case of n = 18. You could win by first dividing by 3.

»
4 года назад, # |
  Проголосовать: нравится +4 Проголосовать: не нравится

Great Contest, u guys are making India Proud :))

»
4 года назад, # |
  Проголосовать: нравится 0 Проголосовать: не нравится

Getting TLE for this (problem E) on case 11......Can someone tell why?

»
4 года назад, # |
  Проголосовать: нравится 0 Проголосовать: не нравится

In problem E "Now we can pick any 1 from this subarray and a −1 either from its left or right to reduce its sum by 1, thus completing the proof" How is this step equivalent to one operation?

  • »
    »
    4 года назад, # ^ |
    Rev. 2   Проголосовать: нравится 0 Проголосовать: не нравится

    Let the index of 1 which we select be i and index of -1 outside the subarray be j.

    Initially:

    s[i] == 1
    s[j] == 0
    t[i] == 0 
    t[j] == 1
    a[i] == 1
    a[j] == -1
    

    We rotate indices i and j for string s. (Doing one operation)

    Result:

    s[i] == 0
    s[j] == 1
    t[i] == 0 
    t[j] == 1
    a[i] == 0
    a[j] == 0
    

    Hence, a[i] == 0 and a[j] == 0 because s[i] == t[i] and s[j] == t[j].

    Now subarray sum has also been decreased by 1 because one of its 1 changed to 0.

»
4 года назад, # |
  Проголосовать: нравится 0 Проголосовать: не нравится

In F you can also root the tree at its centre which ensures the depth of the tree is at most $$$500$$$ and then do a binary search to find the farthest secret node from the root. The second part can be done by querying all nodes which are at a distance of at least $$$mid + 1$$$ and at most $$$high$$$ from the root. If the distance returned by the query is equal to the distance between the two secret nodes, the distance of the farthest secret node lies in $$$[mid + 1, high]$$$; otherwise, it lies in $$$[low, mid]$$$. Here $$$low, mid$$$ and $$$high$$$ are parameters in the binary search.

»
4 года назад, # |
  Проголосовать: нравится +3 Проголосовать: не нравится

The binary search in F2 should be from dis/2 to min(*max_element(dis_tree,dis_tree+N),dis). Because there can be a case where the dis is very small and only making low=(dis/2) would not be enough.

For example when dis=4 and depth=1000, then the low according to editorial will be 2 and high = 1000 thus the queries won't be decreased.

»
4 года назад, # |
  Проголосовать: нравится 0 Проголосовать: не нравится

why do we need task F1?Is there special solution for this problem?

»
4 года назад, # |
  Проголосовать: нравится 0 Проголосовать: не нравится

Can someone please explain why this is failing the 10th test case? I am not able to figure out why. Newbie here. https://codeforces.com/contest/1370/submission/84574756

  • »
    »
    4 года назад, # ^ |
      Проголосовать: нравится 0 Проголосовать: не нравится

    Always stick with meaning of variables and functions. Your check function say 1 if "not greater", so answer is less than equal. So actual value you output should be consistent to this. Perhaps there are other bugs, but fix this one first.

»
4 года назад, # |
  Проголосовать: нравится 0 Проголосовать: не нравится

My solution for 1370E - Сдвиги подпоследовательностей was to add s[i] to the array incor for each s[i] != t[i]. Then break down the values in incor into chunks consisting only of '0's and '1's (e.g. 000, 11, 0). And then count the max length among all chunks(including the one that starts in the end of incor and ends in its beginning), and that would be the answer. (I can provide the proof for this)
But this solution fails on test 10: 84575875
Any ideas why?

  • »
    »
    4 года назад, # ^ |
      Проголосовать: нравится 0 Проголосовать: не нравится

    i also had the same approach. the solution fails for test 10

  • »
    »
    23 месяца назад, # ^ |
      Проголосовать: нравится 0 Проголосовать: не нравится

    try this test: len = 22 ; s = 0000001111101111100001 ; t = not s ; after first rotation first and second chunks of 11111(5) and 11111(5) will be combined to 11111111(8) so the answer is 9

»
4 года назад, # |
Rev. 2   Проголосовать: нравится +1 Проголосовать: не нравится

Here's my approach for E

  • For each operation we would like to build a subsequence made up of alternatives 1's and 0's which are out of their position so that all of them can be corrected in a single move.

  • So we will be searching for alternative 1's and 0's (which are out of their place) to form a subsequence.

  • We are interested in knowing the number of operations, so we will keep a count of "up" and "down" which correspond to number of subsequences we have initialized which need 0->1 and 1->0 as their next element respectively.(s[i]->t[i])

  • Whenever we need to make 1->0 we will decrement "down" and increment "up" (also check down>0 before decrementing). And vice versa for 0->1.

  • At the end sum of up and down will be our answer.

Link for the code

»
4 года назад, # |
  Проголосовать: нравится 0 Проголосовать: не нравится

Can someone please explain the thought process and intuition behind problem E?

  • »
    »
    4 года назад, # ^ |
    Rev. 3   Проголосовать: нравится +3 Проголосовать: не нравится

    Refer to my code.

    Brief explanation:

    We need to convert $$$s$$$ to $$$t$$$. To do this, we are allowed to take any subsequence, and rotate it. We need to find the minimum number of such rotation steps needed to convert $$$s$$$ to $$$t$$$.

    First of all, for it to be possible to convert $$$s$$$ to $$$t$$$, they should have the same number of $$$1$$$ s and $$$0$$$ s. So check this.

    The positions where $$$s[i]=t[i]$$$ don't need to be touched. Now, let's look at the positions where $$$s[i]\ne t[i]$$$. Consider the string: $$$1100$$$. If we rotate this, we get $$$0110$$$. Here, the second position remained $$$1$$$ even after the rotation. So it was useless to include it. We could have instead simply taken the first, third and fourth positions: $$$100$$$, to have the same effect: $$$010$$$. This can in turn be reduced to $$$10$$$ being converted to $$$01$$$.

    Thus, for a move, we'll only take subsequences which have alternating $$$0$$$ s and $$$1$$$ s. The minimum number of moves required will be equal to the minimum number of such subsequences of alternating $$$0$$$ s and $$$1$$$ s that we can obtain.

    To calculate this, we simply iterate over the positions where $$$s[i]\ne t[i]$$$. We keep a count of the number of running subsequences which shall now accept and $$$0$$$ and those which shall now accept a $$$1$$$. Note the the subsequences which shall now accept a $$$0$$$ had their last element as a $$$1$$$, and vice versa.

    While iterating, if $$$s[i]=0$$$, then we shall attach it to a running subsequence, which had its last element as $$$1$$$. After this, this subsequence shall now have its last element as $$$0$$$, and it shall now only accept a $$$1$$$. This decreases the count of the number of running subsequences which accept $$$0$$$ by $$$1$$$, and increases the count of those which accept $$$1$$$ by $$$1$$$. If there are no subsequences which accept $$$0$$$, then we'll simply have to start a new subsequence from this element. The case where $$$s[i]=1$$$ is similar.

    Finally, the answer will be the sum of the number of running subsequences which accept $$$0$$$ and those which accept $$$1$$$.

    • »
      »
      »
      4 года назад, # ^ |
      Rev. 3   Проголосовать: нравится +3 Проголосовать: не нравится

      Thanks. One thing that I was missing to get was that we can attach other shorter length ones to the largest subsequence. Hence only largest pattern of the subsequence matters.

    • »
      »
      »
      4 года назад, # ^ |
        Проголосовать: нравится +3 Проголосовать: не нравится

      Thanks for the explanation, but I'm still a little confused about what you're saying.

      For example I can have the sequence of indices that are 01010 (so the indices expect 10101) but when rotated once it will be 00101 which isn't right. So isn't the condition that the strings that we're generating must be alternating AND also end on a different value from the one that it started on?

      How would keeping track of the number of sequences that accept 0 and 1 take care of this necessary condition?

      • »
        »
        »
        »
        4 года назад, # ^ |
          Проголосовать: нравится +4 Проголосовать: не нравится

        That's a great question.

        Actually, notice that we'll never get this type of subsequence. If we get $$$01010$$$ from $$$s$$$, it means that the corresponding positions at $$$t$$$ are $$$10101$$$. We have already checked that $$$s$$$ and $$$t$$$ have the same number of $$$1$$$ s and $$$0$$$ s. Thus, it means that they have the same number of unmatched $$$1$$$ s and $$$0$$$ s too. Thus, we'll always get even length subsequences. I don't know how to prove this, but it seems correct by intuition to me, as the number of $$$0$$$ s and $$$1$$$ s in $$$s$$$ and $$$t$$$ are same.

    • »
      »
      »
      4 года назад, # ^ |
      Rev. 2   Проголосовать: нравится +3 Проголосовать: не нравится

      Thanks Whiplash99. 1.it's really helpful. 2. I would say better than editorial.:XD

»
4 года назад, # |
  Проголосовать: нравится 0 Проголосовать: не нравится

the second test case of 1370 D is wrong. input : 4 3 1 2 3 4 output: 2

while one can choose subsequence as {2,1,3} giving minimum =1. Please reply fast.

  • »
    »
    4 года назад, # ^ |
      Проголосовать: нравится 0 Проголосовать: не нравится

    you cant shuffle. Subsequence means delete numbers without changing the order.

»
4 года назад, # |
Rev. 5   Проголосовать: нравится 0 Проголосовать: не нравится

FastestFinger

In Problem E

How to rigorously prove that it is always optimal NEVER to take indices $$$i$$$ with $$$s[i]==t[i]$$$? I am struggling with the proof.

»
4 года назад, # |
  Проголосовать: нравится 0 Проголосовать: не нравится
»
4 года назад, # |
  Проголосовать: нравится +3 Проголосовать: не нравится

To prove the greedy method of making a subsequence with target value x in Problem D. Lets prove for even indexes. 2 things to prove:-

If we are on an even index, push any number irrespective of its value to our sub-sequence for the next number. Let the array be a b c d e f. If our subsequence currently has a c, including d will be a c d + odd_subsequence(e f), excluding d will be a c e + odd_subsequence(f). Clearly the former is a better choice since we have a possibility to construct longer subsequence there.

If we are on an odd index, push the nearest number <=x to our subsequence as our next number. let the array a b c d e f g h. If our subsequence currently has a c d, if e <= x and f > x, it is obvious to push e f to our subsequence. Another case will be if e <= x & f <= x, should we think about omitting e ?
If we include e, the subsequence will be a c d e + even_subsequence(f g h), if we exclude e, the subsequence will be a c d f + even_subsequence(g h). Clearly the former is a better choice since we have a possibility to construct longer subsequence there.

»
4 года назад, # |
Rev. 3   Проголосовать: нравится 0 Проголосовать: не нравится

There's another observation for problem F that may offer a solution in even fewer queries, but I'm having trouble finding a bound for its performance. Basically, the idea is that when you do a query during your binary search and find that the result when querying the layer $$$m$$$ is [ $$$v$$$, $$$x$$$] where $$$x > \ell$$$, not only do we know that the target node is higher up than $$$m$$$, we know that the ancestor $$$\frac12(x-\ell)$$$ levels up from $$$v$$$ is a node that is on the path, so we can adjust our lower bound as well!

Since the query answerer must minimize $$$x$$$, it seems like this should actually provide a nontrivial bound improvement, but I was having trouble proving anything or constructing a hard case. My code didn't actually AC until I put this optimization in, so I think my implementation was probably not great (I had the optimization of discarding the largest subtree), but it's possible that this offers a nontrivial optimization. I tried editing the editorial implementation but it still took 11 queries on some tests, and I'm not sure if this is due to implementation. Does anyone have any thoughts?

EDIT: hm, I don't think this offers any improvement since the target node could just be in the bottom layer, never utilizing the insight I described.

»
4 года назад, # |
  Проголосовать: нравится 0 Проголосовать: не нравится

if a number is even and divisible by 4 then after dividing it by its greatest odd divisor it will be of form 2^x x>1 can anyone make me understand this, really not getting it , with proof will help..thanks in advance

  • »
    »
    4 года назад, # ^ |
      Проголосовать: нравится 0 Проголосовать: не нравится

    Consider some number like 360, which is $$$2^3\cdot 3^2 \cdot 5$$$. It is divisible by $$$4=2^2$$$. Now, how do you find its greatest odd divisor? Simple, just take out all the 2's. i.e. $$$3^2 \cdot 5 = 45$$$ must be the greatest divisor (why?). When we divide this number by the greatest odd divisor, that is the same as removing all the odd prime factors. What does that leave us with? Only the 2's. i.e. the number will be of the form $$$2^x$$$. Now, since it was divisible by 4, that means $$$x$$$ must be $$$\geq 2$$$, which is the same thing as saying $$$x > 1$$$.

    • »
      »
      »
      4 года назад, # ^ |
        Проголосовать: нравится 0 Проголосовать: не нравится

      this explanation was really helpful , thank you, i should have thought in this way(^_^).

»
4 года назад, # |
  Проголосовать: нравится 0 Проголосовать: не нравится

Can someone please explain to me why in Problem C, while checking for primes we are only checking divisibility up to 50000?

  • »
    »
    4 года назад, # ^ |
    Rev. 2   Проголосовать: нравится 0 Проголосовать: не нравится

    it is because for all the numbers greater than 50000,u can observe that either the number cant be divisible or it is the number itself. For example consider 1000000000,among all its prime factors at least 1 of them come below 50000 and the other can be found out by dividing this factor from the number.Basically for checking primality,it is enough to go till root n.Hope it answers your question.

  • »
    »
    4 года назад, # ^ |
      Проголосовать: нравится 0 Проголосовать: не нравится

    Ideally it should be sqrt(10^9). 50000 is slightly more. (he should have taken the latter)

    The max factor of a number is sqrt(n), thats why. No need to check after that.

»
4 года назад, # |
Rev. 3   Проголосовать: нравится 0 Проголосовать: не нравится

can someone tell how the check function works for the following test case in problem D? n=5 k=5 arr=4 11 6 10 5 and check of 10 is happening

  • »
    »
    4 года назад, # ^ |
      Проголосовать: нравится +6 Проголосовать: не нравится

    You need to make two sequences for even and for odd. If "good" values on odd, you get 4 11 6 10 5. 4 6 5 are good so they are on odd positions. And for even positions you pick first number you cah get. that's why them 11 and 10 (even if 10 is good number). For even case you pick 4 6 10 5 you pick 4 and 10 on odd positions because you can use anything for them. And, you pick 6 and 5 because they're good and you want to make good on even positions in this sequence. Second sequence is shorter than k so it doesn't work, but first sequence is working, so function should return "yes" because you can make subsequence not exceeding 10.

»
4 года назад, # |
Rev. 2   Проголосовать: нравится 0 Проголосовать: не нравится

Can anyone help me with F1? 84662486. I don't get to know what is the problem in my code that is leading to failure on 9th tc of test set 5

»
4 года назад, # |
  Проголосовать: нравится 0 Проголосовать: не нравится

I need a help with a solution that I have of problem 1370F2 - Загадана пара вершин (сложная версия). I understand the editorial and I know how it works. But I have a solution of my own, which I don't know why should fail. The idea is simple, and quite similar to the one in the editorial.

Let, the hidden nodes are $$$s$$$ and $$$t$$$. Their distance is $$$L$$$. So, let the path between $$$s$$$ and $$$t$$$ be $$$P(s,t)$$$, and $$$|P(s,t)|=L$$$.

  1. QUERY First, query for all nodes, which will give one of the nodes on $$$P(s,t)$$$ and $$$L$$$. Let's call the node $$$x_1$$$.
  2. Root the tree at $$$x_1$$$. Find all the nodes that are $$$min(L,maxDepth)$$$ distance away from $$$x_1$$$.
  3. QUERY query over all these nodes. Here, I hope that a node $$$x_2$$$ is returned where $$$P(x_1,x_2)$$$ contains one of $$$s$$$ or $$$t$$$. Why do I hope that? Without loss of generality, let's say $$$P(x_1,x_2)$$$ contains $$$s$$$. Then $$$dis(s,x_2)+dis(t,x_2)$$$ will be equal to $$$2*dis(s,x_2)+L$$$. As we know $$$L$$$, we can know $$$dis(s,x_2)$$$ and get $$$s$$$ by following the path $$$P(x_2,x_1)$$$.
  4. QUERY As we have found one of the nodes, clearly we can query over all the nodes that are $$$L$$$ distance away from it and find the other node. This will solve the problem in $$$3$$$ queries.

Now, the only thing I need to prove is that in step 3 or query 2, I really get a node $$$x_2$$$ such that $$$P(x_1,x_2)$$$ contains one of $$$s$$$ or $$$t$$$. Let's take the proof out of the context. All I need to know is that in an infinitely large tree, if a path $$$P$$$ consists of nodes $$$p_1, p_2, ...p_L$$$, is it possible to have a node $$$x$$$ which is $$$L$$$ distance away from $$$p_k ($$$ for every $$$1\leq k\leq L)$$$, have minimum $$$dis(p_1,x)+dis(p_L,x)$$$, but not have any of $$$p_1$$$ or $$$p_L$$$ on the path from $$$p_k$$$ to $$$x$$$? I find the idea really simple, and believe that it is correct. Can anyone show me the mistake (or the lack of it)?

  • »
    »
    4 года назад, # ^ |
    Rev. 2   Проголосовать: нравится +3 Проголосовать: не нравится

    This is not correct. On your third query, $$$P(x_1, x_2)$$$ is not guaranteed to contain $$$s$$$ or $$$t$$$. Consider the tree rooted at vertex 1 given by

    1 2
    1 3
    3 4
    3 5
    5 6
    

    Where $$$s=4, t = 2$$$. You will query node $$$6$$$ and get vertex $$$3$$$ as a result. In general when you query a level if the resulting distance is not equal to $$$L$$$ you will only be able to gain information about a node that is on $$$P(s, t)$$$ not $$$s$$$ or $$$t$$$ themselves. This unfortunately does not result in a decrease in overall queries, either.

    • »
      »
      »
      4 года назад, # ^ |
        Проголосовать: нравится -8 Проголосовать: не нравится

      Thanks. When you said, "This is not correct.", I think you mean that this is not correct in a finite tree. Because if the tree was infinite, $$$P(x_1,x_2)$$$ would definitely contain $$$s$$$ or $$$t$$$. In fact, I can prove it right now, just feeling lazy. Thanks again for finding the fault.

»
4 года назад, # |
  Проголосовать: нравится 0 Проголосовать: не нравится

can someone help me why this code for problem E is exceeding its time limit even if its O(n) solution? https://codeforces.com/contest/1370/submission/84710589

  • »
    »
    4 года назад, # ^ |
      Проголосовать: нравится 0 Проголосовать: не нравится

    The way of thinking should be following: if it's indeed O(n) then TL means infinite loop. Otherwise it's not O(n). So, check both: for infinite loop and for "is it really O(n)".

»
4 года назад, # |
  Проголосовать: нравится 0 Проголосовать: не нравится

In problem C, for n=18, Ashishgup: 18/9=2, then FastestFinger : 2-1=1 , then Ashishgup gets 1 and should loose. But answer given is Ashishgup. Please correct me where I am wrong.

»
4 года назад, # |
Rev. 4   Проголосовать: нравится 0 Проголосовать: не нравится

For Div2D, I was wondering if the following approach would work. I tried coding it but it gives WA on TC 22. A small test case for when this fails (could be implementation or idea) would be helpful. Here is the code: 84850363. I create a DSU and each number is a node. I then sort the input array. Then there are two cases: K is even, K is odd.

If K is even, then I need to pick the k/2 smallest numbers such that there is a gap of at least one between each number. I start picking from the smallest number. If there is no conflict (we haven't taken the number before and haven't taken the number after), we can simply take this number. If there is a conflict, then the number is added to that component, and the amount of numbers that can be taken from that component is (size + 1) / 2. We keep taking numbers until the sum of the components with the rule above is >= k/2.

If K is odd, then we need to concern ourselves with the first and last numbers. If we pick the first number or last number, then it can only be a part of the odd indices. If we do not pick either of those numbers, then the sequence can be a part of the even indices, so we must pick k/2 - 1 more.

Please let me know if this idea would work, and if so, what is wrong with the code. Thanks!

»
4 года назад, # |
Rev. 2   Проголосовать: нравится 0 Проголосовать: не нравится

.

»
4 года назад, # |
  Проголосовать: нравится 0 Проголосовать: не нравится

In D, how do we prove that the value found by Binary Search always exists in the array?

»
4 года назад, # |
  Проголосовать: нравится 0 Проголосовать: не нравится

In problem F, how does discarding the largest child reduce queries by 1 ?

»
4 года назад, # |
  Проголосовать: нравится 0 Проголосовать: не нравится

Can anyone explain why I'm getting Runtime error? On problem 1370E - Сдвиги подпоследовательностей

my Submission 86204501

»
4 года назад, # |
  Проголосовать: нравится 0 Проголосовать: не нравится

In problem B, why it is assumed that there are always odd numbers of odds? What if there are even number of odds initially. And If we remove one odd element, we end up having with odd number of odds which won't make 2 as gcd. Can anyone explain?

  • »
    »
    4 года назад, # ^ |
      Проголосовать: нравится 0 Проголосовать: не нравится

    just make pair for (odd,odd) or (even ,even) ,because odd+odd=even ,even+even=even. If there are odd numbers of odds,there are also odd numbers of evens,just discard one odd and one even. If there are even numbers of odds,there are also even numbers of evens,just discard a pair of odds or evens. In short,you need to ensure the 2*n-2 remained array has even odds and even evens.

»
4 года назад, # |
  Проголосовать: нравится 0 Проголосовать: не нравится

If anyone is still stuck here is a detail explanation of D : In this link

»
2 года назад, # |
  Проголосовать: нравится 0 Проголосовать: не нравится

In problem A I think the Time Complexity is O(t) t is the number of testcase

»
12 месяцев назад, # |
  Проголосовать: нравится +1 Проголосовать: не нравится

I have alternate solution for F2 (is similar to solution in the editorial, but its a different way of looking at the problem).

  1. In the first query, query all nodes, to find some node $$$l$$$ on the path between the hidden nodes $$$x$$$ and $$$y$$$.
  2. In the second query, query all nodes at distance $$$ = \lceil \frac{D}{2} \rceil$$$ from $$$l$$$. Let's say the node we find is $$$r$$$. (It's easy to show that we are guaranteed to find some node on the path between $$$x$$$ and $$$y$$$)
  3. Now we will repeat the following query till $$$dis(l, r) < D$$$: Find all nodes at distance $$$ f = \lceil \frac{D - dis(l, r)}{2} \rceil$$$ from $$$l$$$ and $$$r$$$, and are not accessible from either $$$l$$$ or $$$r$$$ without having to cross the other. If the returned node was at the above mentioned distance from $$$l$$$, then set $$$l$$$ to this new node, otherwise if it was at that distance from $$$r$$$, set $$$r$$$ to this new node. (Once again, we are guaranteed to find some node lying on the path between $$$x$$$ and $$$y$$$ in each query.)

Just like the editorial, we find the two hidden nodes in $$$O(\log_2{N})$$$ queries at worst.